loader image
Skip to main content
If you continue browsing this website, you agree to our policies:
x
Completion requirements

Work through the odd-numbered problems 1-37. Once you have completed the problem set, check your answers.

1. (a) derivative of g

(b) derivative of \mathrm{h}

(c) derivative of \mathrm{f}


3. (a) \mathrm{m}_{\mathrm{sec}}=\mathrm{h}-4, \mathrm{~m}_{\tan }=\lim\limits_{h \rightarrow 0} \mathrm{~m}_{\mathrm{sec}}=-4.

(b) \mathrm{m}_{\mathrm{sec}}=\mathrm{h}+1, \mathrm{~m}_{\tan }=\lim\limits_{h \rightarrow 0} \mathrm{~m}_{\mathrm{sec}}=1.


5. (a) \mathrm{m}_{\mathrm{sec}}=5-\mathrm{h}, \mathrm{m}_{\tan }=\lim\limits_{h \rightarrow 0} \mathrm{~m}_{\mathrm{sec}}=5

(b) \mathrm{m}_{\mathrm{sec}}=7-2 \mathrm{x}-\mathrm{h}, \mathrm{m}_{\tan }=\lim\limits_{h \rightarrow 0} \mathrm{~m}_{\mathrm{sec}}=7-2 \mathrm{x}.


7. -1

(b) -1

(c) 0

(d) +1

(e) DNE

(f) DNE


9. \mathrm{f}^{\prime}(\mathrm{x})=\lim\limits_{h \rightarrow 0} \frac{f(x+h)-f(x)}{h}=\lim\limits_{h \rightarrow 0} \frac{\left\{(x+h)^{2}+8\right\}-\left\{x^{2}+8\right\}}{h}=\lim\limits_{h \rightarrow 0} \frac{2 x h+h^{2}}{h}=\lim\limits_{h \rightarrow 0} 2 x+h=2 x \cdot \mathrm{f}^{\prime}(3)=6.


11. \mathrm{f}^{\prime}(\mathrm{x})=\lim\limits_{h \rightarrow 0} \frac{\left\{2(x+h)^{3}-5(x+h)\right\}-\left\{2 x^{3}-5 x\right\}}{h}=\lim\limits_{h \rightarrow 0} \frac{6 x^{2} h+6 x h^{2}+2 h^{3}-5 h}{h}=6 x^{2}-5 . \mathrm{f}^{\prime}(3)=49.


13. For any constant \mathrm{C}, if \mathrm{f}(\mathrm{x})=\mathrm{x}^{2}+\mathrm{C}, then

\mathrm{f}^{\prime}(\mathrm{x})==\lim\limits_{h \rightarrow 0} \frac{f(x+h)-f(x)}{h}=\lim\limits_{h \rightarrow 0} \frac{\left\{(x+h)^{2}+C\right\}-\left\{x^{2}+C\right\}}{h}=\lim\limits_{h \rightarrow 0} \frac{2 x h+h^{2}}{h}=\lim\limits_{h \rightarrow 0} 2 \mathrm{x}+\mathrm{h}=2 \mathrm{x}

The graphs of f(x) = x^2, g(x) = x^2 +3 and h(x) = x^2 – 5 are "parallel" parabolas: g is f shifted up 3 units, and h is f shifted down 5 units.


15. f '(x) = 2x. Then f '(1) = 2 and the equation of the tangent line at (1,9) is y – 9 = 2(x – 1) or y = 2x + 7.        

f '(–2) = – 4 and the equation of the tangent line at (–2,12) is y – 12 = –4(x + 2) or y = –4x + 4.


17. f '(x) = cos(x). Then f '(π) = cos(π) = –1 and the equation of the tangent line at (π,0) is y – 0 = –1(x – π) or y = –x + π

f '(π/2) = cos(π/2) = 0 and the equation of the tangent line at (π/2,1) is y – 1 = 0(x – π/2) or y = 1.


19. (a) y – 5 = 4(x – 2) or y = 4x – 3 (b) x + 4y = 22 or y = –0.25x + 5.5 

(c) f '(x) = 2x so the tangent line is horizontal when x = 0: at the point (0,1).

(d) f '(p) = 2p (the slope of the tangent line) so y – q = 2p(x – p)  or y = 2px + (q – 2p^2)

Since q = p^2 + 1, the equation of the tangent line becomes y = 2px + (p^2 + 1 – 2p^2) = 2px – p^2 + 1

(e) We need p such that –7 = 2p(1) – p2 + 1 or p2 – 2p – 8 = 0. Then p = –2, 4. There are two points with the property we want: (–2, 5) and (4, 17).


21. (a) y ' = 2x, so when x = 1, y ' = 2. Angle = arctan(2) ≈ 1.107 radians ≈ 63°

(b) y ' = 3x^2, so when x = 1, y ' = 3. Angle = arctan(3) ≈ 1.249 radians ≈ 72°

(c) Angle ≈ 1.249 – 1.107 radians = 0.142 radians (or  angle = 72° – 63° = 9°)


23. Graph. On the graph of upward velocity, the units on the horizontal axis are "seconds" and the units on the vertical axis are "feet per second".


25. (a) d(4) = 256 ft. d(5) = 400 ft.  (b) d '(x) = 32x  d '(4) = 128 ft/sec d '(5) = 160 ft/sec.


27. C(x) = √x dollars to produce x golf balls.

Marginal production cost is \mathrm{C}^{\prime}(\mathrm{x})=\frac{1}{2 \sqrt{\mathrm{x}}} \quad dollars per golf ball. \mathrm{C}^{\prime}(25)=\frac{1}{2 \sqrt{25}}=\frac{1}{10} dollars per golf ball. \mathrm{C}^{\prime}(100)=\frac{1}{2 \sqrt{100}}=\frac{1}{20} dollars per golf ball.


29.

(a) \mathrm{A}(0)=0, \mathrm{~A}(1)=1 / 2, \mathrm{~A}(2)=2 and \mathrm{A}(3)=9 / 2

(b) \mathrm{A}(\mathrm{x})=\mathrm{x}^{2} / 2(\mathrm{x} \geq 0)

(c) \frac{\mathrm{d} \mathrm{A}(\mathrm{x})}{\mathrm{dx}}=\mathrm{x}.

(d) \frac{\mathrm{d} \mathrm{A}(\mathrm{x})}{\mathrm{dx}} represents the rate at which \mathrm{A}(\mathrm{x}) is increasing, the rate at which area is accumulating.


31 .

(b) \frac{2}{3 \mathrm{x}^{1 / 3}}

(c) \frac{-4}{x^{5}}

(d) \pi \mathrm{x}^{\pi-1}

(e) 1 if x>-5 and -1 if x


33. f(x)=x^{3}+4 x^{2} (plus any constant)


35.f(x)=5 \cdot \sin (t)


37. f(x)=\frac{1}{2} x^{2}+\frac{1}{3} x^{3}